Download as pdf or txt
Download as pdf or txt
You are on page 1of 4

Homework 3

1. Annual aberration of stars: (a) In class we have derived the aberration formula: tan θ′ = γ1 β+cos sin θ
θ.


Show that this is equivalent to tan θ2 = 1−β θ
1+β tan 2 . (b) Since the Earth’s orbital speed around the
Sun is much smaller than the speed of light, one can take an approximation using β << 1. Show that,
in this approximation, δθ ≡ (θ′ − θ) ∼ = β sin θ. (c)Assuming that a star is at rest relative to the Sun.

If the star locates at an angle θ = 30 above the orbital plane of the Earth. Find the amplitude of the
aberration in annual declination angle variation due to the Earth’s motion around the Sun. (d) If the
star is θ = 30◦ above the equatorial of the Earth, what is the amplitude of the diurnal declination angle
aberration due to Earth’s spin for an observer on the equator? (Note: Please find out the meaning of
”Right ascension” and ”inclination” in astronomical coordinates).
2. Annual aberration of stars: Based on the aberration formula in the previous problem, consider
the observed position of a star over a whole year. (a) Assume that the Earth’s orbit around the Sun is
a exact circle, show that the Star’s apparent position in the sky follows a elliptical trajectory around
its mean position. The long axis of the ellipse is the same for all the stars, called the maximum
aberration. What is the value of the maximum aberration? (c) The size of the short axis depends on
the inclination (latitude) angle (normally called β by astronomer, but it is the same symbol used for
relativistic speed, so we call it η here) of the star relative to the Earth orbit. What is its value?
3. Annual aberration of stars: The first (classical) interpretation of the annual aberration is done
by James Bradley. He measures only the inclinations (latitude) of two stars called γ Draconis and
35 Camelopardalis. The results are shown in the following figure. The mean ecliptic inclination
(inclination related to Earth’s orbital plane) of them are 75.9185◦ and 27.1402◦ , relatively. Based on
this data, find the speed of light.

4. Consider the decay process I → A + B, as shown in the following figure. Note that in the figures, the
symbol p without a capped vector sign represents the magnitude of the 3-momentum. For√example,
pf ≡ |⃗pf |. In the calculation, we also use the symbol E as the relativistic energy, i.e., E = p2 + m2
if the momentum and mass of the particle are p and m, respectively.
In the Center-of-Mass frame, where the initial particle I is at rest, the decayed particles carry momenta
of the same magnitude pf , but at opposite directions, as shown in figure (a). In the lab frame, the
mother particle is moving in x-direction with an initial momentum pi , and the daughter particles are
with momenta pA and pB , with angles θ′ and α′ relative to x-axis, respectively. Based on relativistic
energy-momentum conservation, answer the following questions:

1
(a) Find pf in terms
√ of M , mA , and mB .
(Ans: pf = M 4 + m4A + m4B − 2m2A M 2 − 2m2B M 2 − 2m2A m2B /(2M )).

(b) Show that, if mA = mB = m, then p2f = (M 2 − 4m2 )/4.


(c) Now we turn to the lab frame, as shown in figure (b). If the initial momentum and energy of
the mother particle is known. Show that energy-momentum conservation give three equations for
four variables (pA , pB , θ′ , α′ ), so we can single out one variable as the free variable and express
the others as a function of that variable. A common choice is to use pA (and therefore EA ) as
free variable. Show that:
2EEA − (M 2 + m2A − m2B )
cos θ′ = (1)
2ppA

pB = p2i − 2pi pA cos θ′ + p2A (2)
pA
sin α′ = sin θ′ (3)
pB

(d) What is the maximum value of pA in the lab frame?


(e) In the lab frame, if we do not know the initial momentum and mass of the mother particle, but
measured the momentum of both daughter particles, as well as the angle between them. Let’s
call the angle between the two daughter particles ϕ(= θ′ + α′ ). Express the M and pi in terms of
pA , pB and ϕ. (Ans: M 2 = m2A + m2B + 2EA EB − 2pA pB cos ϕ, and p2i = p2A + p2B + 2pA pB cos ϕ.)
(f) Show that this decay is not possible if M = 0. This implies that photons cannot decay. Note
that you cannot use the CM system in (a) to carry out this proof, because massless particle is
never at rest.
5. Now we consider a collision process, where two particles of the same mass mi collide and produce two
final state particles with the same mass mf . Again we investigate the collision process in two frames,
as shown in the following figures. Figure (a) shows the system in the CM frame. The total momentum
of the system is zero before and after the collision. Figure (b) shows the system in a ”fixed target”
setup, where one of the initial particle is at rest.

2
(a) In fig. (a), find the final state momentum pf in terms of mi ,√mf , and initial momentum pi .
Show that the value of pf is independent from θ. (Ans.: pf = p2i + m2i − m2f ). What is pf if
mi = mf ?
(b) Consider mi = mf = m. Carry out a Lorentz Transformation, such that one of the initial
particle is √
at rest, as shown in Fig. (b). What is the Lorentz factor γ of this transformation?
(Ans. γ = m2 + p2i /mi ). What is p′i ?

(c) Show that, with mi = mf = m, tan θ′ tan α′ = m2 /Ei2 . Here Ei is the energy of the particles in
the CM frame. In the case of high energy collision, m ≪ Ei , show that this result implies that
θ′ = 0 or α′ = 0. On the other hand, in the low energy limit, i.e. pi ≪ Ei , show that this result
implies θ′ + α′ = 90◦ .
(d) What is the total energy of the two initial particles in the collider experiment? What is the
corresponding total energy in the Fixed target experiment? Try to simplify
√ the expression for the
case of mi ≪ pi . Express the results in terms of pi and mi . (Ans: 2 p2i + m2i , 2(p2i + m2i )/mi ,
2pi , 2p2i /mi ). This result shows that for the same collision, doing fixed target experiment requires
much higher energy.
(e) In LHC, two protons two protons with initial energy of 8 TeV collide. Physicists studies the
product of the collision. In this case, since the mass of proton is very much smaller than its
energy, we can apply the condition mi ≪ pi . If we would like to study the same physics as LHC
in a fix-target experiment, what is the energy required for the initial particle?
6. Now we consider the fixed target elastic collision, where a light particle of mass m collide with a
massive particle with mass M at rest, as shown in the following figure. After the collision, the light
particle scatters with momentum pA and angle θ, and the massive particle recoils with momentum pB
and angle α.

in this problem, we assume that m ≪ pi and m ≪ M , so in most cases one can set m = 0 to simplify
the calculation.
(a) Assuming that we know the initial momentum pi , energy-momentum conservation gives us 3
equations, and there are total four unknowns, including pA , pB , θ, and α. Therefore, we have the
freedom to choose one of the variables as free variable, and express all the other variables as a
function of it. The experimental choice is normally θ, because it is the easiest to measure. Show
that
pi M
pA =
pi (1 − cos θ) + M
This equation is used to measure the scattered momentum in this type of experiment.
(b) Take the limit pi << M , the result is approximately pA = pi . Please explain why this is
reasonable.
M
(c) Take the limit pi >> M , show the the result is simplified to pA = 2 sin2 (θ/2)
.

3
7. Now we consider an inelastic collision. High energy photon (γ) proton (p) collision can produce a
neutron (n) plus a pion (π + ) in the process γ + p → n + π + .
(a) For the process to happen, a minimum energy is required for the initial γ + p. Make an argument
that the minimum energy happens when the produced n and π + are both at rest in the center-
of-mass (CM) frame. Therefore, the minimum initial energy is mn + mπ in the CM frame.
(The CM frame is the frame that the total momentum is zero.) What are the 4-momenta of
proton and photon in this case, respectively? Based on this, show that the invariant mass
2 ≡ E2
total − ptotal ) of the minimum energy condition is (mn + mπ ) .
(Minv 2 2

(b) To find the initial minimum energies of the proton and photon in an arbitrary frame, make a
Lorentz boost of the above condition in the direction of the proton’s velocity. In such a system,
the total 4-momentum of the initial state is (Eγ + Ep , p⃗γ + p⃗p ). What is the invariant√
mass of this
system (in terms of Eγ , Ep )? (Recall that, for photon, Eγ = pγ . For proton pp = Ep2 − m2p ).
Since invariant mass is the same in different frames, it should be equal to the result in (a).
(c) If the proton is initially at rest (pp = 0, Ep = mp ), what is the minimum energy of the photon
required to allow this process to happen?
(d) For a fixed small photon energy Eγ ≪ mπ , what is the minimum energy of the proton that allow
this process to happen? Make an argument to show that Ep ≫ mp , and therefore Ep ≃ pp . In
this case, one can simplify the result and give Ep,min = ((mn + mπ )2 − m2p )/(4Eγ ).
(e) High energy proton may collide with the photons of the microwave background (see problem 7
of Homework 2) and produce neutron and pion. What is the minimum proton energy for this
process to happen (give the numerical value in GeV (109 eV))? The energy of cosmic ray protons
are limited by this process, and the energy threshold is called the GZK (Greisen-Zatsepin-Kuzmin)
cutoff. The following figure shows the experimental measurement of the cosmic ray spectrum,
can you identify the GZK cutoff from the data? (Note: mp = 938.27 MeV, mn = 939.57 MeV,
mπ = 139.57 MeV)

You might also like